Đến nội dung

longatk08 nội dung

Có 348 mục bởi longatk08 (Tìm giới hạn từ 08-05-2020)



Sắp theo                Sắp xếp  

#617149 [Trường Xuân toán học miền nam 2016] Vietnam TST 2016 MOCK Test 2

Đã gửi bởi longatk08 on 26-02-2016 - 22:51 trong Thi HSG cấp Tỉnh, Thành phố. Olympic 30-4. Đề thi và kiểm tra đội tuyển các cấp.

Một dạng "chế phẩm" từ bài toán của Long đây, tư tưởng S.O.S rõ ràng dù vẫn hơi động tay, động chân tí:

 

https://artofproblem...4035_inequality




#613915 Tiếp sức bất đẳng thức

Đã gửi bởi longatk08 on 10-02-2016 - 11:46 trong Bất đẳng thức và cực trị

Thứ nhất: Chúng ta có quyền sắp thứ tự hoàn toàn khi vai trò $a,b,c$ trong biểu thức tương đương nhau nhưng với bài này vai trò của các biến không bình đẳng. Thứ 2: Người khác có quyền thắc mắc nếu họ không hiểu về 1 bước trong bài làm của bạn nên tôi nghĩ bạn nên giải thích cho họ hiểu, không hiểu thì phải hỏi, thế thôi. Những bước như thế thì không nên làm tắt. Mà theo biểu thức của bạn gõ ra thì $S_{a},S_{b},S_{c}$ nó đổi chỗ cho nhau hết cả, cái đầu tiên là $S_{c}$, thứ hai phải là $S_{a}$ và thứ 3 là $S_{b}$. Việc xác định đúng các biểu thức hết sức quan trọng trong quá trình bạn biện luận nhé.




#613912 Bất đẳng thức - Cực trị

Đã gửi bởi longatk08 on 10-02-2016 - 11:38 trong Bất đẳng thức và cực trị

Bài 70: Cho $a,b,c$ thực dương thỏa mãn $abc=1$.Chứng minh rằng:

 

$$\frac{a^3c}{a^3+bc+ca}+\frac{b^3a}{b^3+ca+ab}+\frac{c^3b}{c^3+ab+bc}\leq \frac{(a^2+b^2+c^2)^3}{3(a+b+c)^2}$$




#613909 $\sum \frac{1}{a^{2}+ab+b^{2...

Đã gửi bởi longatk08 on 10-02-2016 - 11:32 trong Bất đẳng thức - Cực trị

Bài tương tự: Cho $a,b,c$ không âm thỏa $ab+bc+ac>0$.Chứng minh rằng:

 

$$\frac{1}{a^2+ab+b^2}+\frac{1}{b^2+bc+c^2}+\frac{1}{c^2+ac+a^2}\geq \frac{7(a+b+c)}{3(a+b)(b+c)(c+a)}$$




#613907 Tiếp sức bất đẳng thức

Đã gửi bởi longatk08 on 10-02-2016 - 11:25 trong Bất đẳng thức và cực trị

Bài 43:  Do bận định đăng bài làm trong nháp hôm trước nhưng giờ mời đăng được, mọi người tham khảo:

 

Làm mạnh bđt Cô si:

 

$\frac{a^{4}}{b^{2}}+2ab\geq 3a^{2}+\frac{3}{2}(a-b)^{2}$

 

$\Leftrightarrow (a-b)^{2}\frac{2a^{2}+4ab-3b^{2}}{2b^{2}}\geq 0$

 

Đến đây không phải luôn đúng nên đành chuyển hướng sang S.O.S:

 

tt:  $\frac{b^{4}}{c^{2}}+2bc\geq 3b^{2}+\frac{3}{2}(b-c)^{2}\Leftrightarrow (b-c)^{2}\frac{2b^{2}+4bc-3c^{2}}{2c^{2}}\geq 0$

 

     $\frac{c^{4}}{a^{2}}+2ca\geq 3c^{2}+\frac{3}{2}(c-a)^{2}\Leftrightarrow (c-a)^{2}\frac{2c^{2}+4ac-3a^{2}}{2a^{2}}\geq 0$

 

Cộng vế theo vế ta được:

 

$\frac{a^4}{b^2}+\frac{b^4}{c^2}+\frac{c^4}{a^2}+5(ab+bc+ac)\geq 6(a^2+b^2+c^2)\Leftrightarrow S_{a}(a-b)^{2}+S_{b}(b-c)^{2}+S_{c}(c-a)^{2}\geq 0$

 

Giả sử $a\geq b\geq c\Rightarrow S_{b}> 0$

 

nên chỉ cần CM:   $S_{b}+S_{a};S_{b}+S_{c}\geq 0$ nhưng điều này hiển nhiên đúng.  (Tự nháp)

 

$\Rightarrow ĐPCM$

 

p/s: Ai biết latex bị gì không không dùng được phải dung online

Lời giải này lỗi ở đâu, các bạn thảo luận cho ý kiến tại đây. Các bạn chú ý $S_{a},S_{b},S_{c}$ ở đây là những biểu thức hoàn toàn hoán vị nên về nguyên tác thì ta phải xét 2 trường hợp. Điều này các bạn xem lời giải có thể thấy rõ nên ở đoạn cuối cùng anh Cẩn có xét cả bộ hoán vị của $a,b,c$ để lời giải thêm chặt chẽ.




#613529 Bất đẳng thức - Cực trị

Đã gửi bởi longatk08 on 07-02-2016 - 17:03 trong Bất đẳng thức và cực trị

Bài 67: Cho $a,b,c$ thực dương thỏa mãn $a,b,c \geq 1$ và $\frac{1}{a}+\frac{1}{b}+\frac{1}{c}\leq 2$.Chứng minh rằng:

 

$$\sqrt{a-1}+\sqrt{b-1}+\sqrt{c-1}\geq \frac{9}{4}\sqrt{\frac{3}{abc}}$$




#613434 Tiếp sức bất đẳng thức

Đã gửi bởi longatk08 on 07-02-2016 - 10:32 trong Bất đẳng thức và cực trị

Bài 43:Cho $a,b,c$ thực dương.Chứng minh ta có BĐT sau:

 

$$\frac{a^4}{b^2}+\frac{b^4}{c^2}+\frac{c^4}{a^2}+5(ab+bc+ac)\geq 6(a^2+b^2+c^2)$$




#604861 $\dfrac{a^2}{b}+\dfrac{b^2}...

Đã gửi bởi longatk08 on 23-12-2015 - 16:40 trong Bất đẳng thức - Cực trị

 

 
2) Cho $a,b,c>0$ và $a^6+b^6+c^6=3$. Cmr:
$$\dfrac{a^2}{b}+\dfrac{b^2}{c}+\dfrac{c^2}{a}\ge 3$$
(Võ Quốc Bá Cẩn)
 

 

Tương tự ta cũng có

Cho $a,b,c$ thực dương.Chứng minh BĐT sau:

 

 

$$\frac{a^5}{b}+\frac{b^5}{c}+\frac{c^5}{a}\geq 3\sqrt[3]{\frac{(a^6+b^6+c^6)^2}{9}}$$ (Nguyễn Thúc Vũ Hoàng)




#604858 $\sum \frac{a}{b+c}\geq \frac...

Đã gửi bởi longatk08 on 23-12-2015 - 16:21 trong Bất đẳng thức và cực trị

Bài này có thể giải bằng $SOS$. Ở đây em xin trình bày lại một lời giải bằng $Cauchy \, Schwarz$

$\sum \dfrac{a}{b+c} \geq \dfrac{3\left(a^2+b^2+c^2\right)}{\left(a+b+c\right)^2}+\dfrac{1}{2}$

$\Leftrightarrow \sum \dfrac{a}{b+c} -\dfrac{3}{2} \geq \dfrac{3\left(a^2+b^2+c^2\right)}{\left(a+b+c\right)^2}-1$

$\Leftrightarrow \sum \dfrac{\left(a-b\right)^2}{2\left(b+c\right)\left(c+a\right)} \geq \dfrac{\sum \left(a-b\right)^2}{\left(a+b+c\right)^2}$

-Nếu $a=b=c$ bất đẳng thức đúng

-Nếu $\sum \left(a-b\right)^2 >0$

Áp dụng bất đẳng thức $Cauchy \, Schwarz$ ta có:

$\sum \dfrac{\left(a-b\right)^2}{2\left(b+c\right)\left(c+a\right)}\geq  \dfrac{\left[\sum \left(a-b\right) \right]^2}{2\sum\left(a-b\right)^2\left(b+c\right)\left(c+a\right)}$

Do đó ta chỉ cần chứng minh: $\left(a+b+c\right)^2\left(\sum \left(a-b\right)^2\right) \geq \sum 2\left(b+c\right)\left(c+a\right)\left(a-b\right)^2$

$\Leftrightarrow \sum a^2\left(a-b\right)\left(a-c\right)$(Luôn đúng theo Bất đẳng thức $Schur$ ) 

 Nếu để ý thì bài toán này là 1 bổ đề khá hữu ích, ví dụ khi làm yếu đi BĐT sau:

 

$$\frac{a}{b+c}+\frac{b}{a+c}+\frac{c}{a+b}+\frac{ab+bc+ac}{a^2+b^2+c^2}\leq \frac{5}{2}$$  :mellow:




#604857 $\sum \frac{1}{\sqrt{3a^2+bc}...

Đã gửi bởi longatk08 on 23-12-2015 - 16:18 trong Bất đẳng thức - Cực trị

bài của anh p.k.h được nêu ra trong topic S.O.S và được nêu ra 1 Lần nua tại topic Chia để trị của anh Phan Thành Nam (spam ko z)

Tôi thấy bạn này thích spam nhỉ? Nhiều bài của bạn cứ như post để lấy thành tích hay tự nhiên vô phán 1 câu chả có liên quan gì đến chủ đề topic.... Mong lần sau bạn post bài thì nên đưa ra những bài post mang tính đóng góp hơn nữa.




#602673 $\sum \frac{a}{b+c}\geq \frac...

Đã gửi bởi longatk08 on 11-12-2015 - 20:55 trong Bất đẳng thức và cực trị

Cho $a,b,c$ không âm thỏa mãn $ab+bc+ac>0$.Chứng minh rằng:

 

$$\frac{a}{b+c}+\frac{b}{a+c}+\frac{c}{a+b}\geq \frac{3(a^2+b^2+c^2)}{(a+b+c)^2}+\frac{1}{2}$$

 

Spoiler




#602671 $$\dfrac{a}{b}+\dfrac{b}...

Đã gửi bởi longatk08 on 11-12-2015 - 20:48 trong Bất đẳng thức - Cực trị

Số thực tốt nhất trong trường hợp này nhỏ hơn $1$, bạn trên xem lại kết quả của mình




#602577 $\sum \frac{a^3}{(b+c)^3}+\frac{...

Đã gửi bởi longatk08 on 10-12-2015 - 23:11 trong Bất đẳng thức - Cực trị

Cho $a,b,c$ không âm sao cho $c(a-b) \neq 0$.Chứng minh rằng:

 

$$\sum \frac{a^3}{(b+c)^3}+\frac{8(a+b)^3(b+c)^3(c+a)^3}{(ac-bc)^2(a+b+c)^5}\geq 4$$




#600194 $\frac{ab}{(a+b)^2}+\frac{bc}...

Đã gửi bởi longatk08 on 26-11-2015 - 20:03 trong Bất đẳng thức - Cực trị

Em có thể dùng bổ đề sau:

 

-Với $a,b,c$ không âm thỏa $ab+bc+ac>0$.Ta luôn có:

 

$$\frac{a}{(b+c)^2}+\frac{c}{(a+b)^2}+\frac{b}{(c+a)^2}\geq \frac{4(a^3+b^3+c^3)+15abc}{4(ab+bc+ac)^2}$$

 

Ngoài lề 1 tí: Tình cờ thì BĐT trong bài Mathlinks Contest 2005 có chút "gì đó" liên quan đến bài sau

 

$$\frac{a^2+b^2+c^2}{ab+bc+ac}+\frac{8abc}{(a+b)(b+c)(c+a)}\geq 2+\frac{4(a-b)^2(b-c)^2(c-a)^2}{(a+b)^2(c+a)^2(b+c)^2}$$




#599977 6)$a,b,c >0 ,a+b+c=3, chứng \space minh : \sum{...

Đã gửi bởi longatk08 on 25-11-2015 - 11:28 trong Bất đẳng thức và cực trị

 

2)$a,b,c \in \mathbb{R} chứng\space minh\space 2(1+abc)+\sqrt{2(1+a^2)(1+b^2)(1+c^2) } \geq (1+a)(1+b)(1+c)$

 

Có thể sử dụng C-S như sau:

 

$$\sqrt{2\prod (a^2+1)}=\sqrt{[(a+b)^2+(ab-1)^2][(c+1)^2+(1-c)^2]}\geq (a+b)(c+1)+(ab-1)(1-c)$$




#599831 6)$a,b,c >0 ,a+b+c=3, chứng \space minh : \sum{...

Đã gửi bởi longatk08 on 24-11-2015 - 13:11 trong Bất đẳng thức và cực trị

 

$\leftrightarrow a^{2}b+b^{2}c+c^{2}a+abc \leq 4 $

 

Cho $a,b,c$ không âm thỏa $a+b+c=3$.Chứng minh rằng:

 

$$a^2b+b^2c+c^2a+\frac{abc(3-ab-bc-ac)}{2} \leq 4$$




#599693 $\frac{2a}{b+c}+\frac{2b}{c...

Đã gửi bởi longatk08 on 23-11-2015 - 14:53 trong Bất đẳng thức và cực trị

Cho $a,b,c$ không âm thỏa $ab+bc+ac \neq 0$. Chứng minh rằng:

 

$$\frac{a^2}{(b+c)^2}+\frac{b^2}{(c+a)^2}+\frac{c^2}{(a+b)^2}\geq \frac{3(a^2+b^2+c^2)}{4(ab+bc+ac)}+21\frac{(a-b)^2(b-c)^2(c-a)^2}{(a+b)^2(b+c)^2(c+a)^2} $$




#599691 $\frac{2a}{b+c}+\frac{2b}{c...

Đã gửi bởi longatk08 on 23-11-2015 - 14:44 trong Bất đẳng thức và cực trị

Biến đổi tương đương, bất đẳng thức trở thành :
$(a-b)^2\frac{a^2+b^2+ab+bc+ca}{(a+c)(b+c)(a+b+c)^2}+...\geq 0$ (luôn đúng)

Có chắc rằng BĐT này đúng với mọi $x,y,z$ thuộc R




#599512 $\sqrt{\frac{a+2b}{a+2c}}+\...

Đã gửi bởi longatk08 on 22-11-2015 - 09:07 trong Bất đẳng thức và cực trị

Cho các số thực dương a,b,c. Chứng minh:

$\sqrt{\frac{a+2b}{a+2c}}+\sqrt{\frac{b+2c}{b+2a}}+\sqrt{\frac{c+2a}{c+2b}}\ge 3$ 

Áp dụng AM_GM: $\sqrt{(a+2c)(a+2b)}\leq a+b+c$




#599510 Đề thi chọn đội tuyển dự thi học sinh giỏi quốc gia 2015-2016 tỉnh Nghệ An

Đã gửi bởi longatk08 on 22-11-2015 - 09:00 trong Thi HSG cấp Tỉnh, Thành phố. Olympic 30-4. Đề thi và kiểm tra đội tuyển các cấp.

Cho $a,b,c$ không âm thỏa $ab+bc+ac+abc=4$.Chứng minh rằng:

 

$$a+b+c+\sqrt{a+b+c+2abc+4} \geq 6$$

 

Bất đẳng thức tương tự sau vẫn đúng:

 

$$\frac{a}{b+c}+\frac{b}{c+a}+\frac{c}{a+b}+\left(\sqrt{\frac{a}{b+c}}+\sqrt{\frac{c}{a+b}}+\sqrt{\frac{b}{a+c}}\right)^2\geq 6$$

 

$$(\frac{a}{b+c}+\frac{b}{c+a}+\frac{c}{a+b})\left(\sqrt{\frac{2a}{a+b}}+\sqrt{\frac{2c}{c+a}}+\sqrt{\frac{2b}{b+c}}\right)\geq \frac{9}{2}$$




#599289 $\sum \frac{1}{\sqrt{3a^2+bc}...

Đã gửi bởi longatk08 on 20-11-2015 - 21:19 trong Bất đẳng thức - Cực trị

Cho $a,b,c$ không âm thỏa $ab+bc+ac>0$.Chứng minh rằng:

 

$$\frac{1}{\sqrt{3a^2+bc}}+\frac{1}{\sqrt{3b^2+ac}}+\frac{1}{\sqrt{3c^2+ab}}\geq \frac{2+\sqrt{3}}{\sqrt{3(ab+bc+ac)}} $$

 

Bài toán của anh P.K.Hùng:

 

$$\frac{1}{\sqrt{a^2+bc}}+\frac{1}{\sqrt{b^2+ac}}+\frac{1}{\sqrt{c^2+ab}}\geq \frac{2\sqrt{2}}{\sqrt{ab+bc+ac}}$$




#599249 Chứng minh $x^{2}+y^{2}+z^{2}+xyz\geq...

Đã gửi bởi longatk08 on 20-11-2015 - 17:26 trong Bất đẳng thức và cực trị

$$BĐT\Leftrightarrow 15(x^3+y^3+z^3)+9xyz\geq 9\sum xy(x+y)$$

 

Áp dụng BĐT Schur $x^3+y^3+z^3+3xyz \geq \sum xy(x+y)$ và BĐT AM-GM $x^3+y^3+z^3 \geq x^2y+y^2z+z^2x$




#598704 $\frac{x}{x+z}+\frac{y}{x+y...

Đã gửi bởi longatk08 on 16-11-2015 - 21:38 trong Bất đẳng thức - Cực trị

Chiều ngược lại đúng với $x\geq y\geq z> 0$

Cho $x >=y>=z$ thử với $x=4,y=2,z=1$

 

Ta có BĐT sau xảy ra:

 

$$\frac{x}{x+y}+\frac{y}{y+z}+\frac{z}{z+x}\geq \frac{x+y+z}{x+y+z-\sqrt[3]{xyz}}$$




#598627 $\frac{x}{x+z}+\frac{y}{x+y...

Đã gửi bởi longatk08 on 16-11-2015 - 16:04 trong Bất đẳng thức - Cực trị

Đúng mà bạn!

Chiều ngược lại thì lấy $x=2,y=4,z=3$ là có phản ví dụ




#598539 $\frac{x}{x+z}+\frac{y}{x+y...

Đã gửi bởi longatk08 on 15-11-2015 - 21:30 trong Bất đẳng thức - Cực trị

Thay x=1, y=2, z=3 thì thấy bất đẳng thức trên sai. Phải là $\geq$ thì đúng!

Chiều ngược lại cũng không đúng.